计算高度为$h$的$n$个顶点上(标记和未标记)根树的数目-数学堆栈交换 math.stackexchange.com最新30条 2024-06-16T13:46:58Z https://math.stackexchange.com/feeds/question/1801039 https://creativecommons.org/licenses/by-sa/4.0/rdf https://math.stackexchange.com/q/1801039 4 计算高度为$h的$n$个顶点上(标记的和未标记的)根树的数目$ 用户340082710 https://math.stackexchange.com/users/44711 2016年5月26日T16:22:44Z 2018年10月22日12:24:59Z <p>据我所知,$n$顶点上带标签的根树的数量是$n^{n-1}$。计算高度为$h$的$n$个顶点上(有标签和无标签)根树的数量有已知结果吗?请注意,这些树不需要是二进制的。此外,如果可能的话,我正在寻找一个封闭形式的答案,它可能是$n$和$h$的函数。在最坏的情况下,如果不知道确切结果,我想知道渐近量</p>(第页) https://math.stackexchange.com/questions/1801039/-/1801475#1801475 5 Marko Riedel关于计算高度为$h的$n$个顶点上有根树(标记和未标记)的数目的回答$ 马尔科·里德尔 https://math.stackexchange.com/users/44883 2016年5月26日T23:29:24Z 2018年10月22日下午2:24:59 <p>有了OP提出的问题,我会尽力做推荐人就是这样,提供与OEIS的链接。令人惊讶的是,即使是OEIS在此没有提供通常的各种参考,这表明这个问题是公开的。我们计算生成函数$T_{leh} (z)$</span>对于高度最多为$h$</sspan>,则所需计数为由$T_{\leh}(z)-T_{\leh-1}(z)给出$</span></p><p><P> 我们为标记的组合类<span class=“math-container”>$$\mathcal{T}(T)_{\le h}=\数学{Z}\def\textsc#1{\dosc#1\csod}\def\dosc#1#2\csod{{\rm#1{\small#2}}}\times\textsc{SET}(\mathcal{T}(T)_{\le h-1})$$</span></p><p><P> 对于标记的案例,这转换为$$T_{\le-h}(z)=z\exp T_{\le h-1}(z)$$</p><p>其中$T_{\le0}(z)=0$和$T_{\tle1}(z)=z.$</p><p>例如,使用<span class=“math-container”>$h=3$</span>,我们得到<span class=“math container”>$$T_{\le 3}(z)=z\exp(z\exp(z))$$</span></p><p>从而产生序列<a href=“https://oeis.org/A052512“rel=”nofollow noreferrer“>OEIS A052512</a>:</p><p><span class=“math-container”>$1,2,9,40,205,1176,7399,50576,\ldots$$</p><p>对于<span class=“math-container”>$h=4$</span>,我们得到</p><p>$$T_{\le 4}(z)=z(exp(z\exp(z(z)))$$</p><p>从而产生序列<a href=“https://oeis.org/A052513“rel=”nofollow noreferrer“>OEIS A052513</a>:</p><p>1、2、9、64、505、4536、46249、526352美元</p><p><P> 我们为未标记的组合类<span class=“math-container”>$$\mathcal{T}(T)_{\le h}=\mathcal{Z}\times\textsc{MSET}(\mathcal{T}(T)_{\le h-1})$$</span></p><p>其转化为如下的生成函数,$$T_{\le-h}(z)=z\exp\左(sum{l\ge 1}\压裂{T_{leh-1}(z^l)}{l}\right)$$</span></p><p>其中,$T_{\le 0}(z)=0$和$T_{\ le 1}(z)=z$</p>(第页)<p>我们得到了</p><p>$$T_{le2}(z)=z\exp\left(\sum_{l\ge1}\frac{z^l}{l}\right)=z\exp\log\frac{1}{1-z}=\frac{z}{1-z}$$</span></p><p>这些是根节点,附带有单例。继续我们得到的递归</p><p>$$T_{\le 3}(z)=z\exp\左(sum{l\ge 1}\压裂{1}{l}\sum{q\ge1}z^{ql}\right)=z\exp\左(sum{q\ge 1}\sum{l\ge1}\frac{1}{l}z^{ql}\right)\\=z\exp\sum_{q\ge1}\log\frac{1}{1-z^q}=z\prod_{q\ge1}\frac{1}{1-z^q}$$</span></p><p>从而产生序列<a href=“https://oeis.org/A000041“rel=”nofollow noreferrer“>OEIS A000041</a>:</p><p><span class=“math-container”>$1、1、2、3、5、7、11、15、22、30、42、56美元</p><p>(毫不奇怪,当我们为根和其余部分是一个最多为高度的单扇形分区两个,其中每个节点计数只有一个)</p>(第页)<p><p>从$T_{\le4}(z)$</span>我们得到序列<a href=“https://oeis.org/A001383“rel=”nofollow noreferrer“>OEIS A001383</a>:</p><p>$$1,1,2,4,8,15,29,53,98,177,319,565,\ldots$$</p><p>最后,$T_{\le5}(z)$</span>生成序列<a href=“https://oeis.org/A001384“rel=”nofollow noreferrer“>OEIS A001384</a>:</p><p>1、1、2、4、9、19、42、89、191、402、847美元</p><p>这里有两种解释,这取决于单例应该有零或一个高度。相关链接如下<a href=“https://math.stackexchange.com/questions/1183643/“>MSE链接</a></p><p><P> <strong>备注。</strong>分区数增长的复杂性提供了此问题的难度,即如果不是更困难的话。更详细地研究OEIS条目似乎可以计算出这些疾病的简单复发率</p>(第页)<p><P> 关于标记病例鉴别的复发生产</p><p>$$T'{\leh}(z)=\exp T_{\leh-1}(z)+z\exp T_{leh-1}(z)\times T'_{leh-1}(z)\\=\frac{1}{z}T_{leh}(z)+T_{leh}(z)\乘以T'{leh-1}(x)$$</span></p><p>提取这里的系数</p><p><span class=“math-container”>$$![z^n]T'_{\le h}(z)=T_{\le h,n+1}=n![z^{n+1}]T_{\le h}(z)+n![z^n]T_{\le h}(z)\乘以T'_{\le h-1}(z)\\=frac{1}{n+1}T_{leh,n+1}+n!\和{q=1}^{n}[z^q]T_{\leh}(z)[z^{n-q}]T'{\leh-1}(z)\\=frac{1}{n+1}T_{leh,n+1}+sum_{q=1}^{n}{n选择q}T_{leh,q}T_{leh-1,n-q+1}$$</span></p><p>我们得到了闭合形式</p><p>$$\bbox[5px,border:2px solid#00A000]{T_{leh,n+1}=frac{n+1}{n}\和{q=1}^{n}{n选择q}T_{leh,q}T_}leh-1,n-q+1}。}$$</span></p><p>这里的边界条件是$T_{leh,1}=1$</span>和$T_{0,1}=0.$</p><p><P> 将微分应用于未标记的案例产量</p><p>$$T'{\le-h}(z)=\exp\left(\sum_{l\ge 1}\frac{T_{\le h-1}(z^l)}{l}\right)\\+z\exp\left(\sum_{l\ge1}\frac{T_{leh-1}(z^l)}{l}\right)\时间\sum{l\ge1}T'{leh-1}(z^l)z^{l-1}\\=\frac{1}{z}T_{leh}(z)+T_{\le-h}(z)\times\sum_{l\ge1}T'{\leh-1}(z^l)z^{l-1}$$</span></p><p>提取我们找到的系数</p><p>$$[z^n]T'{\leh}(z)=(n+1)T_{\leh,n+1}\\=[z^{n+1}]T_{leh}(z)+\sum_{l=1}^n\和{q=1}^n[z^q]T_{\le h}(z)[z^{n-q}]z^{l-1}T'{\le h-1}(z^l)\\=T_{\le h,n+1}+\sum_{l=1}^n\和{q=1}^nT_{\le h,q}[z^{n-q-l+1}]T'{\le h-1}(z^l)$$</span></p><p>我们得到了闭合形式</p><p>$$T_{leh,n+1}=\裂缝{1}{n}\sum{l=1}^n\和{q=1}^nT_{\le h,q}[[l|n-q+1]]((n-q+1)/l)T_{\le h-1,(n-q+1)/l}$</span></p><p>它简化为</p><p>$$\bbox[5px,border:2px solid#00A000]{T_{\le h,n+1}=\裂缝{1}{n}\sum{q=1}^n\sum{l|n-q+1}(n-q+1)/l)$$</span></p><p>边界条件与标记的情况相同</p>(第页)<p><P> 这种循环使得计算$T_{le6,n}成为可能$</span>这就产生了</p><p><span class=“math-container”>$1、1、2、4、9、20、47、108、252、582,\\13453086707216121美元</p><p>即<a href=“https://oeis.org/A001385“rel=”nofollow noreferrer“>OEIS A001385和$T_{\le 7,n}$</span>这就产生了</p><p><span class=“math container”>$$1、1、2、4、9、20、48、114、278、676,\\1653、4027、9816、23843美元</p><p>即<a href=“https://oeis.org/A034823“rel=”nofollow noreferrer“>OEIS A034823</a></p><p><P> 实现这两个循环的Maple代码如下</p>(第页)<预>电话:=过程(h,m)选项记忆;局部n;如果m=1,则如果h=0,则返回0,否则返回1 fi-fi;n:=m-1;(n+1)/个*加(二项式(n,q)*T(h,q)*T(h-1,n-q+1),q=1..n);结束;X(X):=过程(h,m)选项记忆;局部n;如果m=1,则如果h=0,则返回0,否则返回1 fi-fi;n:=m-1;1/n个*加(加(X(h,q)*&#96;如果&#96;(n-q+1模l=0,(n-q+1)/l*X(h-1,(n-q+1)/l),0),q=1…n),l=1..n);结束;</pre>